Skip to main content
Log in

FTAP in finite discrete time with transaction costs by utility maximization

  • Published:
Finance and Stochastics Aims and scope Submit manuscript

Abstract

The aim of this paper is to prove the fundamental theorem of asset pricing (FTAP) in finite discrete time with proportional transaction costs by utility maximization. The idea goes back to L.C.G. Rogers’ proof of the classical FTAP for a model without transaction costs. We consider one risky asset and show that under the robust no-arbitrage condition, the investor can maximize his expected utility. Using the optimal portfolio, a consistent price system is derived.

This is a preview of subscription content, log in via an institution to check access.

Access this article

Price excludes VAT (USA)
Tax calculation will be finalised during checkout.

Instant access to the full article PDF.

Similar content being viewed by others

Notes

  1. Given \(X_{T} \in\mathcal{A}^{0}_{T}\) such that E[U(X T )]=∞, we set E[U(X T )]:=−∞.

References

  1. Barron, E.N., Cardaliaguet, P., Jensen, R.: Conditional essential suprema with applications. Appl. Math. Optim. 48, 229–253 (2003)

    Article  MATH  MathSciNet  Google Scholar 

  2. Castaing, C., Valadier, M.: Convex Analysis and Measurable Multifunctions. Lecture Notes in Math., vol. 580. Springer, Berlin (1977)

    MATH  Google Scholar 

  3. Campi, L., Owen, M.P.: Multivariate utility maximization with proportional transaction costs. Finance Stoch. 15, 461–499 (2011)

    Article  MathSciNet  Google Scholar 

  4. Cvitanić, J., Karatzas, I.: Hedging and portfolio optimization under transaction costs: a martingale approach. Math. Finance 6, 113–165 (1996)

    Google Scholar 

  5. Czichowsky, C., Muhle-Karbe, J., Schachermayer, W.: Transaction costs, shadow prices, and duality in discrete time. SIAM J. Financ. Math. 5, 258–277 (2014)

    Article  MathSciNet  Google Scholar 

  6. Dalang, R.C., Morton, A., Willinger, W.: Equivalent martingale measures and no-arbitrage in stochastic securities market models. Stoch. Stoch. Rep. 29, 185–201 (1990)

    Article  MATH  MathSciNet  Google Scholar 

  7. Delbaen, F., Schachermayer, W.: The Mathematics of Arbitrage. Springer Finance. Springer, Berlin (2006)

    MATH  Google Scholar 

  8. Deelstra, G., Pham, H., Touzi, N.: Dual formulation of the utility maximization problem under transaction costs. Ann. Appl. Probab. 11, 1353–1383 (2001)

    Article  MATH  MathSciNet  Google Scholar 

  9. Harrison, J.M., Kreps, D.M.: Martingales and arbitrage in multi-period securities markets. J. Econ. Theory 20, 381–408 (1979)

    Article  MATH  MathSciNet  Google Scholar 

  10. Harrison, J.M., Pliska, S.R.: Martingales and stochastic integrals in the theory of continuous trading. Stoch. Process. Appl. 11, 215–260 (1981)

    Article  MATH  MathSciNet  Google Scholar 

  11. Kabanov, Y., Rásonyi, M., Stricker, Ch.: No-arbitrage criteria for financial markets with efficient friction. Finance Stoch. 6, 371–382 (2002)

    Article  MATH  MathSciNet  Google Scholar 

  12. Kabanov, Y., Rásonyi, M., Stricker, Ch.: On the closedness of sums of convex cones in L 0 and the robust no-arbitrage property. Finance Stoch. 7, 403–411 (2003)

    Article  MATH  MathSciNet  Google Scholar 

  13. Kabanov, Y., Safarian, M.: Markets with Transaction Costs—Mathematical Theory. Springer Finance. Springer, Berlin (2010)

    Book  Google Scholar 

  14. Pennanen, T.: Dual representation of superhedging costs in illiquid markets. Math. Financ. Econ. 5, 233–248 (2011)

    Article  MATH  MathSciNet  Google Scholar 

  15. Rásonyi, M., Stettner, L.: On utility maximization in discrete-time financial market models. Ann. Appl. Probab. 15, 1367–1395 (2005)

    Article  MATH  MathSciNet  Google Scholar 

  16. Rásonyi, M.: New methods in the arbitrage theory of financial markets with transaction costs. In: Donati-Martin, C., Émery, M., Rouault, A., Stricker, Ch. (eds.) Séminaire de Probabilités XLI. Lecture Notes in Mathematics, vol. 1934, pp. 455–462. Springer, Berlin (2008)

    Chapter  Google Scholar 

  17. Rásonyi, M.: Arbitrage with transaction costs revisited. In: Delbaen, F., Rásonyi, M., Stricker, Ch. (eds.) Optimality and Risk—Modern Trends in Mathematical Finance, pp. 211–226. Springer, Berlin (2009)

    Chapter  Google Scholar 

  18. Rogers, L.C.G.: Equivalent martingale measures and no-arbitrage. Stoch. Stoch. Rep. 51, 41–49 (1994)

    Article  MATH  Google Scholar 

  19. Rokhlin, D.B.: A theorem on martingale selection for relatively open convex set-valued random sequences. Math. Notes 81, 543–548 (2007)

    Article  MATH  MathSciNet  Google Scholar 

  20. Rokhlin, D.B.: Constructive no-arbitrage criterion under transaction costs in the case of finite discrete time. Theory Probab. Appl. 52, 93–107 (2008)

    Article  MATH  MathSciNet  Google Scholar 

  21. Schachermayer, W.: A Hilbert space proof of the fundamental theorem of asset pricing in finite discrete time. Insur. Math. Econ. 11, 249–257 (1992)

    Article  MATH  MathSciNet  Google Scholar 

  22. Schachermayer, W.: The fundamental theorem of asset pricing under proportional transaction costs in finite discrete time. Math. Finance 14, 19–48 (2004)

    Article  MATH  MathSciNet  Google Scholar 

Download references

Acknowledgements

The authors thank the Deutsche Forschungsgemeinschaft (DFG) for financial support.

Author information

Authors and Affiliations

Authors

Corresponding author

Correspondence to Jörn Sass.

Appendix

Appendix

We want to prepare the proof of Lemma 3.3. Given two sub-σ-fields \(\mathcal{A}_{0} \subset\mathcal{A}_{1}\) and \(0 < \underline{X}_{0} \le\overline{X}_{0}\) \(\mathcal{A}_{0}\)-measurable as well as \(0 < \underline{X}_{1} \le\overline{X}_{1}\) \(\mathcal{A}_{1}\)-measurable, we assume that

$$\mathbf{L}^0\big({-}K(\underline{X}_0, \overline{X}_0), \mathcal{A}_0\big) \cap\mathbf{L}^0\big(K(\underline{X}_1, \overline{X}_1), \mathcal {A}_1\big) $$

is a vector space. It is straightforward to check that under this assumption

$$\big\{ \mathbf {E}\big[ |\underline{X}_1 - \overline{X}_0 | \,\big\vert \, \mathcal{A}_0 \big] = 0 \big\} = \big\{ \mathbf {E}\big[ |\underline{X}_0 - \overline{X}_1 | \,\big\vert \,\mathcal{A}_0 \big] = 0 \big\} , $$

and \(A:=\{ \mathbf {E}[ |\underline{X}_{1} - \overline{X}_{0} | \,\vert\, \mathcal {A}_{0} ] = 0 \}\) is the biggest \(\mathcal{A}_{0}\)-measurable set such that

$$\underline{X}_0=\overline{X}_0=\underline{X}_1=\overline{X}_1 \quad\hbox{on}\ A. $$

For ωA, we define P 0(ω) to be the orthogonal projection on the linear space generated by ; for ωA, we set P 0(ω):=0. Then we have P 0 v=v iff \(v \in\mathbf{L}^{0}(-K(\underline{X}_{0}, \overline{X}_{0}), \mathcal{A}_{0}) \cap\mathbf{L}^{0}(K(\underline{X}_{1}, \overline{X}_{1}), \mathcal{A}_{1})\).

The following lemma is a version of Proposition 3.3 from [15]. We write \(\ell^{X_{1}}(v)\) for the liquidation value of v with respect to the bid and ask prices \(\underline{X}_{1}, \overline {X}_{1}\), i.e.

$$\ell^{X_1}(v) = \begin{cases} v^0 + v^1 \underline{X}_1, & v^1 \ge0, \\ v^0 + v^1 \overline{X}_1, & v^1 \le0. \end{cases} $$

The liquidation value is continuous in v and satisfies (αv)=αℓ(v) for α≥0. This is why the proof from [15] also works (with the obvious changes) in our model with transaction costs.

Lemma A.1

If \(\mathbf{L}^{0}(-K(\underline{X}_{0}, \overline{X}_{0}), \mathcal{A}_{0}) \cap\mathbf{L}^{0}(K(\underline{X}_{1}, \overline{X}_{1}), \mathcal{A}_{1})\) is a vector space, then there exists a strictly positive \(\mathcal {A}_{0}\)-measurable γ 0 such that

$$\mathbf {P}[ \ell^{X_1} (v) < -\gamma_0 \,\vert\, \mathcal{A}_0 ] > 0 $$

for every \(v \in\mathbf{L}^{0}( -K (\underline{X}_{0}, \overline{X}_{0}), \mathcal{A}_{0})\) which satisfies P 0 v=0 and |v|=1.

Proof of Lemma 3.3

For t=0,1, we pick an \(\mathcal{A}_{t}\)-measurable \(S_{t} \in \operatorname {ri}[\underline{X}_{t}, \overline{X}_{t}]\) and set for α t ∈(0,1)

$$\underline{V}_t := (1-\alpha_t) S_t + \alpha_t \underline{X}_t, \qquad \overline{V}_t := (1-\alpha_t) S_t + \alpha_t \overline{X}_t. $$

We fix α 1∈(0,1) such that \(2 (1-\alpha_{1}) (1+\overline {X}_{1}^{2}) < \gamma_{0}/2\). It follows from

and

for every |v|=1 that

$$\mathbf {P}[\ell^{V_1}(v) < -\gamma_0/2 \,\vert\, \mathcal{A}_0 ] \ge \mathbf {P}[\ell ^{X_1}(v) < - \gamma_0 \,\vert\, \mathcal{A}_0] > 0 $$

for every \(v \in\mathbf{L}^{0} (-K(\underline{X}_{0}, \overline{X}_{0}), \mathcal{A}_{0})\) which satisfies P 0 v=0 and |v|=1.

We have \(v - P_{0}v \in\mathbf{L}^{0} (-K(\underline{V}_{0}, \overline {V}_{0}), \mathcal{A}_{0})\) and \(\ell^{V_{1}}(v)=\ell^{V_{1}}(v-P_{0}v)\) for every \(v \in\mathbf{L}^{0} (-K(\underline{V}_{0}, \overline{V}_{0}), \mathcal {A}_{0})\). Thus, it is enough to find an appropriate α 0 such that

$$\mathbf {P}[\ell^{V_1}(v) < 0 \,\vert\, \mathcal{A}_0] > 0 \quad\hbox{on}\ \{v \neq0 \} $$

for every \(v \in\mathbf{L}^{0} (-K(\underline{V}_{0}, \overline{V}_{0}), \mathcal{A}_{0})\) with P 0 v=0. We can assume that |v|=1 on {v≠0}. It is straightforward to check that then

where \(c(\alpha_{0}):=(1/\alpha_{0} - 1) (1 + S_{0}^{2})\). Note that (id−P 0)(vc(α 0)e 0)≠0 a.s. and c(α 0)↓0 as α 0↑1. Now

$$\ell^{V_1}\bigg( \frac{v - c(\alpha_0)e^0}{|(\operatorname {id}-P_0) (v - c(\alpha_0)e^0)|} \bigg) = \ell^{V_1}\bigg( \frac{(\operatorname {id}-P_0)(v - c(\alpha_0)e^0)}{|(\operatorname {id}-P_0) (v - c(\alpha_0)e^0)|} \bigg) <-\gamma_0/2 $$

implies on {v≠0} that

$$\ell^{V_1} (v) \le-(\gamma_0/2) \big(1-|(\operatorname {id}- P_0)c(\alpha _0)e^0|\big) + c (\alpha_0). $$

We fix α 0 near 1 such that the last expression is strictly negative. It follows that

$$\mathbf {P}[\ell^{V_1}(v) < 0 \,\vert\, \mathcal{A}_0] > 0 \quad\hbox{on}\ \{v \neq0 \} $$

for every \(v \in\mathbf{L}^{0} (-K(\underline{V}_{0}, \overline{V}_{0}), \mathcal{A}_{0})\) with P 0 v=0. □

Proof of Lemma 3.5

Claim (a). We show that on \(A=\{ \operatorname {inf}_{\mathcal{H}} \underline{X} =\mathbf {E}_{\mathbf{Q}} [ Z \,\vert\, \mathcal{H} ] \}\), we have \(\inf_{\mathcal{H}} \underline{X} = \sup_{\mathcal{H}} \overline{X}\). Indeed, from \(\operatorname {inf}_{\mathcal{H}} \underline{X} \le\underline{X} \le Z\), we get that \(\operatorname {inf}_{\mathcal{H}} \underline{X} = \underline{X} = Z\) and thus \(\underline{X}=\overline{X}\) on A, since \(Z \in \operatorname {ri}[ \underline{X}, \overline{X} ]\). Then \(\operatorname {inf}_{\mathcal{H}} \underline{X}= \overline{X}\) and \(\operatorname {inf}_{\mathcal{H}} \underline{X} = \operatorname {sup}_{\mathcal{H}} \overline{X}\) on A. Similarly, we get that \(\inf_{\mathcal{H}} \underline{X}\) and \(\sup_{\mathcal{H}} \overline{X}\) coincide on \(\{ \mathbf {E}_{\mathbf{Q}}[ Z \,\vert\, \mathcal{H} ] = \operatorname {sup}_{\mathcal {H}} \overline{X} \} \).

Claim (b). For \(n \in\mathbb{N}\), put

$$\begin{aligned} \tilde{a}_n &:= \frac{\mathbf{1}_{ \{ \underline{X} < \operatorname {inf}_\mathcal{H} \underline{X} + \frac{1}{n} \}}}{\mathbf {P}[ \underline{X} < \operatorname {inf}_\mathcal{H} \underline{X} + \frac{1}{n} \,\vert\, \mathcal{H}] } + \frac{1}{n} \frac {\mathbf{1}_{ \{ \underline{X} \ge \operatorname {inf}_\mathcal{H} \underline{X} + \frac{1}{n} \}}}{1+\underline{X}}, \\ \tilde{b}_n &:= \frac{\mathbf{1}_{ \{ \overline{X} > f_n \}}}{\mathbf {P}[ \overline{X} > f_n \,\vert\, \mathcal{H}] } + \frac{1}{n} \frac {\mathbf {1}_{ \{ \overline{X} \le f_n \}}}{1+\overline{X}}, \end{aligned}$$

where

$$f_n := \mathbf{1}_ {\{ \operatorname {sup}_{\mathcal{H}} \overline{X} < \infty\}} \bigg(\operatorname {sup}_{\mathcal{H}} \overline{X} - \frac{1}{n}\bigg) + 1_{ \{ \operatorname {sup}_{\mathcal{H} } (\overline{X} ) =\infty\} } n. $$

Then \(\tilde{a}_{n}, \tilde{b}_{n} > 0\) and \(\mathbf {E}[\tilde{a}_{n} \underline{X} \,\vert\, \mathcal{H}] \rightarrow \operatorname {inf}_{\mathcal{H}} \underline{X}\), \(\mathbf {E}[\tilde{b}_{n} \overline{X} \,\vert\, \mathcal{H}] \rightarrow \operatorname {sup}_{\mathcal{H}} \overline{X} \ \)a.s. as n→∞. Since \(1 \le \mathbf {E}[\tilde{a}_{n} \,\vert\, \mathcal{H}] \le1 + \frac{1}{n}\) and \(1 \le \mathbf {E}[\tilde{b}_{n} \,\vert\, \mathcal{H}] \le1 + \frac{1}{n}\), we normalize \(\tilde{a}_{n}\) and \(\tilde{b}_{n}\), i.e. replace \(\tilde{a}_{n}\) by \(\frac{\tilde{a}_{n}}{\mathbf {E}[ \tilde{a}_{n} \,\vert\, \mathcal{H} ]}\) and \(\tilde{b}_{n}\) by \(\frac{\tilde{b}_{n}}{\mathbf {E}[ \tilde{b}_{n} \,\vert\, \mathcal {H} ]}\), and still have \(\mathbf {E}[\tilde{a}_{n} \underline{X} \,\vert\, \mathcal {H}] \to \operatorname {inf}_{\mathcal{H}} \underline{X}\), \(\mathbf {E}[\tilde{b}_{n} \overline {X} \,\vert\, \mathcal{H}] \rightarrow \operatorname {sup}_{\mathcal{H}} \overline{X} \ \)a.s. as n→∞.

Now we set \(\varOmega_{1} := \{ \operatorname {inf}_{\mathcal{H}} \underline{X} < \operatorname {sup}_{\mathcal{H}} \overline{X} \}\) and \(\varOmega_{2} := \{ \operatorname {inf}_{\mathcal{H}} \underline{X} = \operatorname {sup}_{\mathcal{H}} \overline{X} \}\); thus Ω=Ω 1Ω 2. For ωΩ 1 (up to a null set), we choose \(n=n( \omega) \in\mathbb{N}\) minimal such that \((\operatorname {inf}_{\mathcal{H}} \underline{X}) ( \omega) \le \mathbf {E}[\tilde{a}_{n} \underline{X} \,\vert\, \mathcal{H}](\omega) < c (\omega)\). Then ωn(ω) is \(\mathcal{H}\)-measurable on Ω 1, and \(\tilde{a}\) defined as \(\tilde{a} ( \omega) := \tilde{a}_{n (\omega)} (\omega)\) on Ω 1 and \(\tilde{a} ( \omega) = 1\) on Ω 2 is strictly positive and satisfies \(\mathbf {E}[\tilde{a} \,\vert \, \mathcal{H}] = 1\), \(\mathbf {E}[\tilde{a} \underline{X} \,\vert\, \mathcal {H}] < c \ \mbox{on}\ \varOmega_{1}\). Similarly, \(\tilde{b}\) defined by \(\tilde{b}(\omega) := \tilde{b}_{m ( \omega)} ( \omega)\) on Ω 1, where \(m( \omega) \in\mathbb{N}\) is minimal such that \(\mathbf {E}[ \tilde{b}_{m} \overline{X} \,\vert\, \mathcal{H} ] ( \omega)> c( \omega)\), and \(\tilde{b} ( \omega) = 1\) on Ω 2 is strictly positive and satisfies \(\mathbf {E}[\tilde{b} \,\vert\, \mathcal{H} ] = 1\), \(\mathbf {E}[ \tilde{b} \overline{X} \,\vert\, \mathcal{H}] > c \ \mbox{on}\ \varOmega_{1}\). We put

$$\begin{aligned} a& := \frac{c-\mathbf {E}[ \tilde{a} \underline{X} \,\vert\, \mathcal{H} ]}{\mathbf {E}[\tilde{b} \overline{X} \,\vert\, \mathcal{H} ] - \mathbf {E}[ \tilde{a} \underline{X} \,\vert\, \mathcal{H} ]} \tilde{b} \mathbf {1}_{\varOmega_1} + \frac{1}{2} \mathbf{1}_{\varOmega_2}, \\ b&:= \frac{\mathbf {E}[\tilde{b} \overline{X} \,\vert\, \mathcal{H}]-c}{\mathbf {E}[\tilde{b} \overline{X} \,\vert\, \mathcal{H}] - \mathbf {E}[ \tilde{a} \underline {X} \,\vert\, \mathcal{H}]} \tilde{a} \mathbf{1}_{\varOmega_1}+\frac{1}{2} \mathbf {1}_{\varOmega_2}. \end{aligned}$$

Then we have \(\mathbf {E}_{\mathbf{Q}} [Z \,\vert\, \mathcal{H} ] = c\), where \(Z:=\frac{a\underline{X}+b\overline{X}}{a+b} \in \operatorname {ri}[\underline{X},\overline{X}]\) and Q is defined by \(\frac {\mathrm{d} \mathbf{Q}}{\mathrm{d}\mathbf {P}} := a+b\). □

Proof of Lemma 3.6

Closedness of F follows from the fact that φ(0)≤φ(ta) for aF, t>0, due to concavity. Fix a closed \(K \subset \{x \in\mathbb{R}^{d} \, : \, |x|=1\}\) and choose a sequence \((x_{j})_{j=1}^{\infty}\) of \(\mathcal{H}\)-measurable random variables with values in K such that \((x_{j}(\omega))_{j=1}^{\infty}\) is dense in KC(ω) on the event {KC≠∅}. Then

$$\{{ \omega\in\varOmega}:{ F(\omega) \cap K \neq\emptyset}\} = \bigcup_{M=1}^{\infty} \bigcap_{k=1}^{\infty} \bigcup_{j = 1}^{\infty} \bigl\{ {\omega\in\varOmega}:{\varphi\big(\omega, kx_j(\omega )\big) \ge-M }\bigr\} \in\mathcal{H}. $$

Closedness of A is clear. Now, for a fixed \(K \subset\mathbb{R}^{d}\) which is supposed to be compact, we choose a sequence of \(\mathcal {H}\)-measurable random variables \((y_{j})_{j=1}^{\infty}\) with values in K such that \((y_{j}(\omega))_{j=1}^{\infty}\) is dense in KC(ω) on the event {KC≠∅}. Further, let \((c_{n})_{n=1}^{\infty}\) be a sequence of \(\mathcal{H}\)-measurable random variables such that \((c_{n}(\omega))_{n=1}^{\infty}\) is dense in C(ω). Then

$$\begin{aligned} &\{{ \omega\in\varOmega}:{ A (\omega) \cap K \neq\emptyset}\} \\ &\quad= \bigcap_{m=1}^{\infty} \bigcup_{j = 1}^{\infty} \bigcap_{n=1}^{\infty} \biggl\{ {\omega\in\varOmega}\,:\,{ \varphi\big(\omega, y_j(\omega )\big) \ge\varphi \big(\omega, c_n(\omega) \big) - \frac{1}{m} }\biggr\} \in\mathcal{H}. \end{aligned}$$

Now, by straightforward measurable selection arguments (see Theorem III.6 in [2] and use the fact that in \(\mathbb{R}^{d}\), every open set is a union of compact sets), α and β can be defined in an \(\mathcal{H}\)-measurable way. □

Remark A.2

We can find an equivalent measure P′ such that \(\mathbf {E}_{\mathbf{P}^{\prime}} [U(\ell(v)) \,\vert\, \mathcal{F}_{t-1}]\) and \(\mathbf{E}_{\mathbf{P}^{\prime}} [U^{\prime}(\ell(v)) \,\vert\, \mathcal {F}_{t-1}]\) are finite for every \(v \in\mathbf{L}^{0} (\mathbf{R}^{2}, \mathcal{F}_{t-1})\). Further, we can find a version of \(\mathbf {E}_{\mathbf{P}^{\prime}} [U(\ell(a)) \,\vert\, \mathcal{F}_{t-1}]\) for every \(a \in\mathbb{R}^{2}\) such that for every scenario ωΩ, the mapping \(a \mapsto\mathbf{E}_{\mathbf{P}^{\prime}} [U(\ell (a)) \,\vert\, \mathcal{F}_{t-1}](\omega)\) is finite and continuous.

To see this, we choose a function \(g : \mathbb{R}_{+} \rightarrow (0,\infty)\) which is decreasing and continuous such that

$$\vert U(a \cdot x) \vert g(\vert x \vert) \le \frac{1}{g(\vert a \vert)} + c \quad\forall a, x \in\mathbb{R}^d, $$

where c is a fixed constant. If U is bounded from above by 0, g can be defined by g(t):=(max{1,|U(−t 2)|})−1 (see [18]). Denote by P′ the equivalent probability measure with density

and denote by k the regular conditional P-distribution for \(( \underline{V}_{t}, \overline{V}_{t} )\) given \(\mathcal{F}_{t-1}\). Then for any \(v \in\mathbf{L}^{0} (\mathbb{R}^{2}, \mathcal{F}_{t-1} )\), we have by disintegration

with . Using the estimate from above plus dominated convergence, the claim follows easily.  □

Rights and permissions

Reprints and permissions

About this article

Check for updates. Verify currency and authenticity via CrossMark

Cite this article

Sass, J., Smaga, M. FTAP in finite discrete time with transaction costs by utility maximization. Finance Stoch 18, 805–823 (2014). https://doi.org/10.1007/s00780-014-0241-z

Download citation

  • Received:

  • Accepted:

  • Published:

  • Issue Date:

  • DOI: https://doi.org/10.1007/s00780-014-0241-z

Keywords

Mathematics Subject Classification (2010)

JEL Classification

Navigation